Difference between revisions of "2004 AMC 12A Problems/Problem 16"

m (Solution 2)
 
(8 intermediate revisions by 2 users not shown)
Line 8: Line 8:
 
<math>\textbf {(A) } 0\qquad \textbf {(B) }2001^{2002} \qquad \textbf {(C) }2002^{2003} \qquad \textbf {(D) }2003^{2004} \qquad \textbf {(E) }2001^{2002^{2003}}</math>
 
<math>\textbf {(A) } 0\qquad \textbf {(B) }2001^{2002} \qquad \textbf {(C) }2002^{2003} \qquad \textbf {(D) }2003^{2004} \qquad \textbf {(E) }2001^{2002^{2003}}</math>
  
== Solution ==
+
== Solution 1 ==
 
For all real numbers <math>a,b,</math> and <math>c</math> such that <math>b>1,</math> note that:
 
For all real numbers <math>a,b,</math> and <math>c</math> such that <math>b>1,</math> note that:
 
<ol style="margin-left: 1.5em;">
 
<ol style="margin-left: 1.5em;">
Line 23: Line 23:
 
from which <math>c=\boxed{\textbf {(B) }2001^{2002}}.</math>
 
from which <math>c=\boxed{\textbf {(B) }2001^{2002}}.</math>
  
~Azjps (Fundamental Logic)
+
~Azjps ~MRENTHUSIASM
  
~MRENTHUSIASM (Reconstruction)
+
== Solution 2 ==
 +
Let
 +
<cmath>\begin{align*}
 +
x &= 2001^a, \\
 +
a &= 2002^b, \\
 +
b &= 2003^c, \\
 +
c &= 2004^d.
 +
\end{align*}</cmath>
 +
It follows that <cmath>x = 2001^{2002^{2003^{2004^d}}}.</cmath>
 +
The smallest value of <math>x</math> occurs when <math>d\rightarrow -\infty,</math> so this expression becomes
 +
<cmath>x = 2001^{2002^{2003^0}} = 2001^{2002^1} = \boxed{\textbf {(B) }2001^{2002}}.</cmath>
  
 
==Video Solution (Logical Thinking)==
 
==Video Solution (Logical Thinking)==

Latest revision as of 02:01, 23 January 2023

Problem

The set of all real numbers $x$ for which

\[\log_{2004}(\log_{2003}(\log_{2002}(\log_{2001}{x})))\]

is defined is $\{x\mid x > c\}$. What is the value of $c$?

$\textbf {(A) } 0\qquad \textbf {(B) }2001^{2002} \qquad \textbf {(C) }2002^{2003} \qquad \textbf {(D) }2003^{2004} \qquad \textbf {(E) }2001^{2002^{2003}}$

Solution 1

For all real numbers $a,b,$ and $c$ such that $b>1,$ note that:

  1. $\log_b a$ is defined if and only if $a>0.$
  2. $\log_b a>c$ if and only if $a>b^c.$

Therefore, we have \begin{align*} \log_{2004}(\log_{2003}(\log_{2002}(\log_{2001}{x}))) \text{ is defined} &\implies \log_{2003}(\log_{2002}(\log_{2001}{x}))>0 \\ &\implies \log_{2002}(\log_{2001}{x})>1 \\ &\implies \log_{2001}{x}>2002 \\ &\implies x>2001^{2002}, \end{align*} from which $c=\boxed{\textbf {(B) }2001^{2002}}.$

~Azjps ~MRENTHUSIASM

Solution 2

Let \begin{align*} x &= 2001^a, \\ a &= 2002^b, \\ b &= 2003^c, \\ c &= 2004^d. \end{align*} It follows that \[x = 2001^{2002^{2003^{2004^d}}}.\] The smallest value of $x$ occurs when $d\rightarrow -\infty,$ so this expression becomes \[x = 2001^{2002^{2003^0}} = 2001^{2002^1} = \boxed{\textbf {(B) }2001^{2002}}.\]

Video Solution (Logical Thinking)

https://youtu.be/46c-VN1QzWk

~Education, the Study of Everything

See also

2004 AMC 12A (ProblemsAnswer KeyResources)
Preceded by
Problem 15
Followed by
Problem 17
1 2 3 4 5 6 7 8 9 10 11 12 13 14 15 16 17 18 19 20 21 22 23 24 25
All AMC 12 Problems and Solutions